Algebras generated as commutative algebras.

  • Thread starter Thread starter Kreizhn
  • Start date Start date
Click For Summary
A commutative R-algebra S is finitely generated as an algebra over R if and only if it is finitely generated as a commutative algebra over R. The distinction lies in the nature of the generators: in a non-commutative setting, the variables do not commute, affecting the structure of the generated algebra. The discussion highlights the need to demonstrate the equivalence of these two notions when S is commutative. Participants express uncertainty about the definitions and implications of these algebraic structures. Clarifying the relationship between these forms of generation is essential for solving the problem.
Kreizhn
Messages
714
Reaction score
1

Homework Statement


Let R be a commutative ring. Prove that a commutative R-algebra S is finitely generated as an algebra over R if and only if it is finitely generated as a commutative algebra over R.

The Attempt at a Solution


I'm really not even sure what this is asking. I know that for an R-algebra S, we can view it as being generated as an R-module and as an R-algebra, and that they do not generally agree. However, what's the difference between being generated as an R-algebra and being generated as a commutative R-algebra?
 
Physics news on Phys.org
Hi Kreizhn! :smile:

I'm not sure, but I'll have a go at it. I think that S is finitely generated as a commutative A-algebra if there is a surjection

A[T_1,...,T_n]\rightarrow S

However, S is finitely generated as an A-algebra if there is a surjection

A<T_1,...,T_n>\rightarrow S

Where A<T_1,...,T_n> are polynomials in n variables, but the variables don't commute. So we don't have T_1T_2=T_2T_1.

What you need to show is that if S is commutative, then these two notions are equivalent.
 
Okay thanks. I'll give it a shot and see what I can come up with.
 
Question: A clock's minute hand has length 4 and its hour hand has length 3. What is the distance between the tips at the moment when it is increasing most rapidly?(Putnam Exam Question) Answer: Making assumption that both the hands moves at constant angular velocities, the answer is ## \sqrt{7} .## But don't you think this assumption is somewhat doubtful and wrong?

Similar threads

  • · Replies 8 ·
Replies
8
Views
2K
  • · Replies 6 ·
Replies
6
Views
2K
  • · Replies 3 ·
Replies
3
Views
824
Replies
1
Views
1K
  • · Replies 1 ·
Replies
1
Views
2K
  • · Replies 3 ·
Replies
3
Views
977
  • · Replies 5 ·
Replies
5
Views
2K
  • · Replies 2 ·
Replies
2
Views
1K
  • · Replies 7 ·
Replies
7
Views
1K
  • · Replies 5 ·
Replies
5
Views
895